Calc 2 Homework: Find Series Solutions to 2 Problems

In summary, Infinite Series Calculus 2 homework problemThe Infinite Series Calculus 2 homework problem asks for the power series representation for ln(1+x²). The student got divergent at both endpoints and did not use a valid counterexample.
  • #1
gokugreene
47
0
Infinite Series Calculus 2 homework problem

Hello. I need help finding the answer on two of these.

#1. Find a power series representation for [tex]f(x)=ln(1+x^2)[/tex]

I have
[tex]f'(x)=\frac{(2x)}{(1+x^2)}[/tex]
So I factor out 2x and have [tex]\frac{1}{(1+x^2)}[/tex] which is a geometric series =
[tex]\sum(-1)^{n}x^{2n}[/tex] then multiply 2x times the series
2x * [tex]\sum(-1)^{n}x^{2n}[/tex] = [tex]\sum(-1)^{n}2x^{2n+1}[/tex]

Then, [tex]\int\sum(-1)^{n}2x^{2n+1}[/tex] = [tex] \sum\frac{(-1)^{n+1}2x^{2n+2}}{2n+2}[/tex]

Did I do that right??

Ok Problem #2 Find the radius of convergence and interval of convergence of the series.
[tex]\sum\frac{(-2)^{n}(x+3)^{n}}{\sqrt{n}}[/tex]
for R I get 1/2 and I am having trouble figuring out whether the endpoints (5/2)<x<(7/2), are convergent.

Thanks!
 
Last edited:
Physics news on Phys.org
  • #2
I'm not sure what you're doing:

[tex]f(x) = \sum _{n=0} ^{\infty } \frac{f^{(n)}(0)}{n!}x^n[/tex]

what you're doing looks nothing like this, so unless I made a mistake or am missing something, what you did looks wrong. Here's a quick check:

ln(1 + 1²) = ln(2) = 0.69314718055994530941723212145818 ~ 0.69

On the other hand:

[tex] \sum\frac{(-1)^{n+1}2(1)^{2n+2}}{2n+2}[/tex]

[tex] = \sum\frac{(-1)^{n+1}}{n+1}[/tex]

[tex] = -\frac{1}{1} + \frac{1}{2} - \frac{1}{3} + \frac{1}{4} < 0 < 0.69[/tex]

so that's definitely wrong.
 
  • #3
Ok say I want to find the power series representation for ln(5-x)
it will be equal to [tex]ln(5)-\sum\frac{x^{n+1}}{(n+1)5^{n+1}}[/tex]
 
  • #4
i think

I think what you did on the first one is correct, or at least i got the same result.
I think AKG counterexample is not valid, because the power series you gave:
[tex]f(x)=ln(1+x^2)=\sum\frac{(-1)^{n+1}2x^{2n+2}}{2n+2}[/tex]
only converges for -1<x<1 (This comes from the interval of convergence of the geometric where you started from).

Problem 2: Your endpoints are wrong. The power series converges absolutely for -7/2<x<-5/2 and you should check for x=-7/2 and x=-5/2. For one you should use leibnitz and for the other one is just a p-series.

Maybe i am saying something stupid, because i don't know a lot about math.
I hope this helps, Paul.
 
  • #5
Well if you take x very close to 1, but less than 1, then your series will still be negative. On the other hand ln(1 + x²) will still be close to 0.69.
 
  • #6
pbialos said:
I think what you did on the first one is correct, or at least i got the same result.
I think AKG counterexample is not valid, because the power series you gave:
[tex]f(x)=ln(1+x^2)=\sum\frac{(-1)^{n+1}2x^{2n+2}}{2n+2}[/tex]
only converges for -1<x<1 (This comes from the interval of convergence of the geometric where you started from).

Problem 2: Your endpoints are wrong. The power series converges absolutely for -7/2<x<-5/2 and you should check for x=-7/2 and x=-5/2. For one you should use leibnitz and for the other one is just a p-series.

Maybe i am saying something stupid, because i don't know a lot about math.
I hope this helps, Paul.

I found the series to be divergent at both endpoints. Is that what you got pbialos?

AKG is he right about the first one?

Thanks for the help
 
  • #7
I haven't touched series in over a year, but the formula I gave you in the second post does give the McLaurian expansion (which is just the Taylor expansion). If that's what you're doing (if that's what you mean by power series) then what you're doing looks nothing like what I've seen before. Also, if pbialos is right in that the interval of convergence is (-1,1) then the modification of my counteraxmple which I gave in post 5 should hold. Plug in x=1 and you'll see that they're very unequal: the series is negative and ln(1 + x²) = ln(2) is approximately 0.69. If 1 is not a valid counterexample, then it's still pretty clear that x = 0.9999 will give ln(1 + x²) close to 0.69 (at very least, it will be positive) and the series will still be negative.
 
  • #8
correction

For the second one, i got divergent when x=-7/2 and convergent for x=-5/2.
On the first one, i am not sure, maybe AKG or someone else can show us what is wrong with your procedure. I am just studying this subject myself, and i am not really sure about a lot of things.


Paul.
 
  • #9
How did you get convergent for x=-5/2?
[tex]\frac{(-2)^{n}(-\frac{1}{2})^{n}}{\sqrt{n}}[/tex] = [tex]\frac{(-1)^n}{\sqrt{n}}[/tex] is a divergent p-series p<1
 
  • #10
I'm not sure what you're procedure is. What does your textbook say? Have you seen a formula like this?:

[tex]f(x) = \sum _{n=0} ^{\infty } \frac{f^{(n)}(0)}{n!}x^n[/tex]

I don't know if there are various power series expansions of functions, but the Taylor series expansion does express f(x) as a power series, and it is given by the formula above. Actually, the above is a special case where you expand about 0, since it's easiest to do so, but in general, the expansion is given by:

[tex]f(x) = \sum _{n=0} ^{\infty } \frac{f^{(n)}(a)}{n!}(x-a)^n[/tex]

Could you tell me how you're getting what you're getting, and why you're doing it. What process does your book give you? Why are you factoring out 2x? Why aren't you computing higher order derivatives, and why is there no "n!" in your summand?
 
  • #11
no

[tex]\sum\frac{(-1)^n}{\sqrt{n}}[/tex] is not a p-series, it is an alternating series. You should use the alternating series test.
 
  • #12
gokugreene said:
Then, [tex]\int\sum(-1)^{n}2x^{2n+1}[/tex] = [tex] \sum\frac{(-1)^{n+1}2x^{2n+2}}{2n+2}[/tex]

I you integrate the series term-wise correctly, you get:

[tex]\int\sum_{n=0}^\infty (-1)^{n}2x^{2n+1} = \sum_{n=0}^\infty \frac{(-1)^n 2x^{2n+2}}{2n+2} + C=\sum\frac{(-1)^n x^{2n+2}}{n+1} + C[/tex]
So your answer had the wrong sign and you had to account for the constant of integration (which you can find by plugging in x=0 for example).

You could also get it from the power series of ln(1+x) (gotten by integrating 1/(1+x)):

[tex]\ln(1+x)=\sum_{n=0}^\infty \frac{(-1)^n x^{n+1}}{n+1}[/tex]

So replacing x by x^2:

[tex]\ln(1+x^2)=\sum_{n=0}^\infty \frac{(-1)^n x^{2n+2}}{n+1}[/tex]
 
  • #13
Thanks I see my mistakes!
 

1. What is the purpose of finding series solutions in Calc 2 homework?

In Calc 2, we use series solutions to approximate functions that cannot be expressed in a simple form. This allows us to solve problems that would otherwise be difficult or impossible using traditional methods.

2. How do you determine the convergence of series solutions?

The convergence of series solutions can be determined by using tests such as the ratio test, root test, or alternating series test. These tests help us determine whether the series will converge or diverge.

3. Can series solutions be used for any type of function?

No, series solutions are typically used for functions that are infinitely differentiable. This means that the function must be able to be expressed as a power series, with infinitely many terms.

4. What is the process for finding series solutions to a problem?

To find series solutions, we typically start by assuming a power series representation for the function. Then, we use this representation to find the coefficients of the series using differentiation and substitution. Finally, we determine the convergence of the series to ensure the solution is valid.

5. Are there any common mistakes to avoid when finding series solutions?

One common mistake is forgetting to check for convergence after finding the series representation. Another mistake is using the wrong tests to determine convergence, leading to incorrect solutions. It is also important to be careful with algebraic manipulations when finding coefficients, as a small mistake can change the entire solution.

Similar threads

  • Calculus and Beyond Homework Help
Replies
1
Views
231
  • Calculus and Beyond Homework Help
Replies
3
Views
401
  • Introductory Physics Homework Help
Replies
8
Views
1K
  • Introductory Physics Homework Help
3
Replies
95
Views
4K
Replies
2
Views
772
  • Introductory Physics Homework Help
Replies
8
Views
394
  • Introductory Physics Homework Help
Replies
15
Views
242
  • Introductory Physics Homework Help
Replies
28
Views
352
  • Introductory Physics Homework Help
Replies
19
Views
664
  • Introductory Physics Homework Help
Replies
11
Views
567
Back
Top